Convergence of an alternating series : $ sum_{ngeq 1} frac{(-1)^n|sin n|}{n}$












18












$begingroup$



Study the convergence of $$displaystyle sum_{ngeq 1} frac{(-1)^n|sin n|}{n}.$$




I am stuck with this series, we need probably some measure of irrationally of $pi$, unfortunately I am unfamiliar with this. So here is my attempt :



Let $f(x) = sum frac{|sin{n}|}{n} x^n, |x| < 1$



It's not difficult to compute the Fourier series of $|sin(x)|$ :



$$
displaystyle|sin(x)|=frac{2}{pi}-frac{4}{pi}sum_{n=1}^{+infty}frac{cos(2nx)}{4n^2-1}
$$



Then Fubini's theorem (Series Version) works very well (because the previous series converges absolutely at $x$ fixed ) and all calculations made, we find that for all $xin( -1,1)$:



$$
displaystyle f(x)=frac{2}{pi}sum_{n=1}^{+infty}frac{x^n}{n}-frac{4}{pi}sum_{p=1}^{+infty}frac{x^2-2xcos(p)}{(4p^2-1)(x^2-2xcos(p)+1)}
$$



However, the second sum I have not been able to show the convergence. I feel the series diverge because the following series
$$
displaystylesumfrac{1}{p^2sin^2left(frac{p}{2}right)}
$$



diverge because $0$ is an accumulation point of $displaystyle (nsin(n))$ sequence.



Any ideas (for the original series) ?










share|cite|improve this question











$endgroup$








  • 3




    $begingroup$
    artofproblemsolving.com/Forum/viewtopic.php?p=393755#p393755
    $endgroup$
    – Random Variable
    Apr 5 '14 at 17:38










  • $begingroup$
    The series is not alternating.
    $endgroup$
    – Did
    Jan 12 '18 at 13:48
















18












$begingroup$



Study the convergence of $$displaystyle sum_{ngeq 1} frac{(-1)^n|sin n|}{n}.$$




I am stuck with this series, we need probably some measure of irrationally of $pi$, unfortunately I am unfamiliar with this. So here is my attempt :



Let $f(x) = sum frac{|sin{n}|}{n} x^n, |x| < 1$



It's not difficult to compute the Fourier series of $|sin(x)|$ :



$$
displaystyle|sin(x)|=frac{2}{pi}-frac{4}{pi}sum_{n=1}^{+infty}frac{cos(2nx)}{4n^2-1}
$$



Then Fubini's theorem (Series Version) works very well (because the previous series converges absolutely at $x$ fixed ) and all calculations made, we find that for all $xin( -1,1)$:



$$
displaystyle f(x)=frac{2}{pi}sum_{n=1}^{+infty}frac{x^n}{n}-frac{4}{pi}sum_{p=1}^{+infty}frac{x^2-2xcos(p)}{(4p^2-1)(x^2-2xcos(p)+1)}
$$



However, the second sum I have not been able to show the convergence. I feel the series diverge because the following series
$$
displaystylesumfrac{1}{p^2sin^2left(frac{p}{2}right)}
$$



diverge because $0$ is an accumulation point of $displaystyle (nsin(n))$ sequence.



Any ideas (for the original series) ?










share|cite|improve this question











$endgroup$








  • 3




    $begingroup$
    artofproblemsolving.com/Forum/viewtopic.php?p=393755#p393755
    $endgroup$
    – Random Variable
    Apr 5 '14 at 17:38










  • $begingroup$
    The series is not alternating.
    $endgroup$
    – Did
    Jan 12 '18 at 13:48














18












18








18


14



$begingroup$



Study the convergence of $$displaystyle sum_{ngeq 1} frac{(-1)^n|sin n|}{n}.$$




I am stuck with this series, we need probably some measure of irrationally of $pi$, unfortunately I am unfamiliar with this. So here is my attempt :



Let $f(x) = sum frac{|sin{n}|}{n} x^n, |x| < 1$



It's not difficult to compute the Fourier series of $|sin(x)|$ :



$$
displaystyle|sin(x)|=frac{2}{pi}-frac{4}{pi}sum_{n=1}^{+infty}frac{cos(2nx)}{4n^2-1}
$$



Then Fubini's theorem (Series Version) works very well (because the previous series converges absolutely at $x$ fixed ) and all calculations made, we find that for all $xin( -1,1)$:



$$
displaystyle f(x)=frac{2}{pi}sum_{n=1}^{+infty}frac{x^n}{n}-frac{4}{pi}sum_{p=1}^{+infty}frac{x^2-2xcos(p)}{(4p^2-1)(x^2-2xcos(p)+1)}
$$



However, the second sum I have not been able to show the convergence. I feel the series diverge because the following series
$$
displaystylesumfrac{1}{p^2sin^2left(frac{p}{2}right)}
$$



diverge because $0$ is an accumulation point of $displaystyle (nsin(n))$ sequence.



Any ideas (for the original series) ?










share|cite|improve this question











$endgroup$





Study the convergence of $$displaystyle sum_{ngeq 1} frac{(-1)^n|sin n|}{n}.$$




I am stuck with this series, we need probably some measure of irrationally of $pi$, unfortunately I am unfamiliar with this. So here is my attempt :



Let $f(x) = sum frac{|sin{n}|}{n} x^n, |x| < 1$



It's not difficult to compute the Fourier series of $|sin(x)|$ :



$$
displaystyle|sin(x)|=frac{2}{pi}-frac{4}{pi}sum_{n=1}^{+infty}frac{cos(2nx)}{4n^2-1}
$$



Then Fubini's theorem (Series Version) works very well (because the previous series converges absolutely at $x$ fixed ) and all calculations made, we find that for all $xin( -1,1)$:



$$
displaystyle f(x)=frac{2}{pi}sum_{n=1}^{+infty}frac{x^n}{n}-frac{4}{pi}sum_{p=1}^{+infty}frac{x^2-2xcos(p)}{(4p^2-1)(x^2-2xcos(p)+1)}
$$



However, the second sum I have not been able to show the convergence. I feel the series diverge because the following series
$$
displaystylesumfrac{1}{p^2sin^2left(frac{p}{2}right)}
$$



diverge because $0$ is an accumulation point of $displaystyle (nsin(n))$ sequence.



Any ideas (for the original series) ?







real-analysis sequences-and-series






share|cite|improve this question















share|cite|improve this question













share|cite|improve this question




share|cite|improve this question








edited Apr 4 '14 at 16:32

























asked Apr 1 '14 at 15:10







user119228















  • 3




    $begingroup$
    artofproblemsolving.com/Forum/viewtopic.php?p=393755#p393755
    $endgroup$
    – Random Variable
    Apr 5 '14 at 17:38










  • $begingroup$
    The series is not alternating.
    $endgroup$
    – Did
    Jan 12 '18 at 13:48














  • 3




    $begingroup$
    artofproblemsolving.com/Forum/viewtopic.php?p=393755#p393755
    $endgroup$
    – Random Variable
    Apr 5 '14 at 17:38










  • $begingroup$
    The series is not alternating.
    $endgroup$
    – Did
    Jan 12 '18 at 13:48








3




3




$begingroup$
artofproblemsolving.com/Forum/viewtopic.php?p=393755#p393755
$endgroup$
– Random Variable
Apr 5 '14 at 17:38




$begingroup$
artofproblemsolving.com/Forum/viewtopic.php?p=393755#p393755
$endgroup$
– Random Variable
Apr 5 '14 at 17:38












$begingroup$
The series is not alternating.
$endgroup$
– Did
Jan 12 '18 at 13:48




$begingroup$
The series is not alternating.
$endgroup$
– Did
Jan 12 '18 at 13:48










4 Answers
4






active

oldest

votes


















11





+50







$begingroup$

By Dirichlet's convergence test, this series will converge if we can show that there exists a constant $C$ such that $$left|sum_{nleq x}(-1)^{n}|sin(n)|right|leq C$$ for all $x$.



Lets write $$sum_{nleq x}(-1)^{n}|sin(n)|=sum_{nleqfrac{x}{2}}|sin(2n)|-sum_{nleqfrac{x+1}{2}}|sin(2n-1)|.$$ Then for $x=2N$, an even number, Euler Maclaurin summation yields $$sum_{nleq N}|sin(2n)|=int_{1}^{N}|sin(2t)|dt+sum_{k=1}^{K}frac{(-1)^{k}}{k!}B_{k}left(frac{d^{k-1}}{dt^{k-1}}|sin(2t)|biggr|_{t=1}^{t=N}right)$$ $$ -frac{(-1)^{K}}{K!}int_{1}^{N}B_{K}({t})left(frac{d^{k}}{dt^{k}}|sin(2t)|right)dt.$$ Note that $|sin(x)|$ has infinitely many derivatives everywhere except at integer multiples of $pi$, and so the above holds for any $K>0$. Since $$|B_{k}({x})|leq k!2^{1-k}pi^{-k}zeta(k),$$ and since the derivatives of $|sin(t)|$ are bounded in absolute value by $1$, it follows that $$left|sum_{nleq N}|sin(2n)|-int_{1}^{N}|sin(2t)|dtright|leq4sum_{k=1}^{K}frac{zeta(k)}{(2pi)^{k}}+frac{2zeta(K)N}{(2pi)^{K}}.$$ The series $sum_{k=1}^{infty}frac{zeta(k)}{(2pi)^{k}}$ converges absolutely, so by taking $K=N$ we see that there exists a constant $C_{1}$ such that $$left|sum_{nleq N}|sin(2n)|-int_{1}^{N}|sin(2t)|dtright|leq C_{1}$$ for all $N$. Similarly, there exists a constant $C_{2}$ such that $$left|sum_{nleq N}|sin(2n-1)|-int_{1}^{N}|sin(2t-1)|dtright|leq C_{2}.$$ Thus by the triangle inequality, $$left|sum_{nleq x}(-1)^{n}|sin(n)|right|leq C_{1}+C_{2}+left|int_{1}^{N}|sin(2t)|dt-int_{1}^{N}|sin(2t-1)|dtright|$$



$$leq C_{1}+C_{2}+int_{N-1/2}^{N}|sin(2t)|dt+int_{1}^{3/2}|sin(2t-1)|dt$$



$$leq C_{3}$$ for some constant $C_{3}$. This implies the desired result.






share|cite|improve this answer









$endgroup$









  • 2




    $begingroup$
    Euler Maclaurin summation formula might not work for functions with discontinuous derivatives.
    $endgroup$
    – i707107
    Apr 25 '17 at 1:23










  • $begingroup$
    See my answer to the same problem: math.stackexchange.com/questions/2202138/… This uses different idea, but it is quite a standard approach for such series.
    $endgroup$
    – i707107
    Apr 25 '17 at 1:31












  • $begingroup$
    See en.wikipedia.org/wiki/Euler%E2%80%93Maclaurin_formula for the notations I use, and for the case $p=1$, $S=sum_{m<jleq n} f(j)$, $I=int_m^n f(x) dx$, $$ S-I=frac{B_1}{1!}(f(n)-f(m)) + R_1$$ $$=frac{f(n)-f(m)}2 +int_m^n left( {x}-frac12right) df(x)$$ This is integration by parts for Riemann-Stiltjes integral. A bound for the remainder $$ |R_1|leq frac 2{2pi} int_m^n |f'(x)| dx,$$ in case $f'$ is continuous. But, it is $$ |R_1|leq frac 2{2pi} int_m^n |df(x)| $$ if $f'$ has discontinuity. The function $f(t)=|sin t|$ has a discontinuous derivative.
    $endgroup$
    – i707107
    Jan 9 at 0:06





















12












$begingroup$

The series converges. It is enough to show that the sequence of the following partial sums converges:
begin{align}
s_N &= sum_{n=1}^{N} left(frac{ |sin 2n|}{2n}-frac{|sin (2n+1)|}{2n+1}right)=sum_{n=1}^N left(frac{(2n+1)|sin 2n|- 2n|sin (2n+1)| }{2n(2n+1)} right)\
&=sum_{n=1}^N left( frac{|sin 2n|-|sin (2n+1)|}{2n+1}+frac{|sin 2n|}{2n(2n+1)}right).
end{align}

Thus, it is enough to show that the following converges:
$$
S_N = sum_{n=1}^N frac{|sin 2n|-|sin (2n+1)|}{2n+1}.
$$

We consider a partition of $mathbb{N}$ into four disjoint sets $A_{1}$, $A_{2}$, $A_{3}$, $A_{4}$ defined by:
$$
A_{1}={ninmathbb{N}: sin 2n >0, sin (2n+1)>0}, A_{2}={ninmathbb{N}: sin 2n >0, sin (2n+1)<0},
$$

$$
A_{3}={ninmathbb{N}: sin 2n <0, sin (2n+1)>0}, A_{4}={ninmathbb{N}: sin 2n <0, sin (2n+1)<0}.
$$

Note that
$$
A_{1}={ninmathbb{N}: 2n mathrm{mod} 2pi in (0,pi-1)}, A_{2}={ninmathbb{N}: 2n mathrm{mod} 2pi in (pi-1,pi)},$$

$$
A_{3}={ninmathbb{N}: 2n mathrm{mod} 2pi in (-1,0)}, A_{4}={ninmathbb{N}: 2n mathrm{mod} 2pi in (-pi, -1)}.$$

By trigonometric identities,
$$
nin A_{1} Longrightarrow |sin 2n|-|sin (2n+1)| = sin 2n - sin(2n+1) = -2cos(2n+frac12)sin frac12, $$

$$
nin A_{2} Longrightarrow |sin 2n|-|sin (2n+1)| = sin 2n + sin(2n+1) = 2sin(2n+frac12)cos frac12, $$

$$
nin A_{3} Longrightarrow |sin 2n|-|sin (2n+1)| = -sin 2n - sin(2n+1) = -2sin(2n+frac12)cosfrac12, $$

$$
nin A_{4} Longrightarrow |sin 2n|-|sin (2n+1)| = -sin 2n + sin(2n+1) = 2cos(2n+frac12)sin frac12.$$

We define
$$
f_1(x)=-I_{(0,pi-1)}(x)2cos(x+frac12)sinfrac12, f_2(x)=I_{(pi-1,pi)}(x)2sin(x+frac12)cosfrac12,$$

$$
f_3(x)=-I_{(-1,0)}(x)2sin(x+frac12)cosfrac12, f_4(x)=I_{(-pi,-1)}(x)2cos(x+frac12)sinfrac12 $$

where $I_A$ is the characteristic function of $A$. Note that these functions $f_i(x)$ are of bounded variation in $[-pi,pi]$. Thus, $f=f_1+f_2+f_3+f_4$ is of a bounded variation.



We need Koksma's inequality p. 143, Theorem 5.1 of 'Uniform Distribution of Sequences' by Kuipers and Niederreiter:




Theorem [Koksma]



Let $f$ be a function on $I=[0,1]$ of bounded variation $V(f)$, and suppose we are given $N$ points $x_1, ldots , x_N$ in $I$ with discrepancy
$$
D_N:=sup_{0leq aleq bleq 1} left|frac1N #{1leq nleq N: x_n in (a,b) } -(b-a)right|.
$$

Then
$$
left|frac1N sum_{nleq N} f(x_n) - int_I f(x)dx right|leq V(f)D_N.
$$




To control the discrepancy, we apply Erdos-Turan inequality p. 112, Theorem 2.5 of Kuipers and Niederreiter's book.




Theorem[Erdos-Turan]



Let $x_1, ldots, x_N$ be $N$ points in $I=[0,1]$. Then there is an absolute constant $C>0$ such that for any positive integer $m$,
$$
D_Nleq C left( frac1m+ sum_{h=1}^m frac1h left| frac1Nsum_{n=1}^N e^{2pi i h x_n}right|right).
$$




The sequence of our interest is $x_n = 2n$ mod $2pi$. The above two inequalities together applied to $f=f_1+f_2+f_3+f_4$ yields the following: There is an absolute constant $C>0$ such that for any positive integer $m$,
$$
left|frac1N sum_{nleq N} f(x_n) - frac1{2pi} int_{-pi}^{pi} f(x)dx right|leq C left( frac1m+ frac1Nsum_{h=1}^m frac1{hlangle frac h{pi} rangle}right).
$$

A result on the irrationality measure of $pi$ by Salikhov implies that
$$
left| frac1{pi} - frac pq right| geq frac 1{q^{mu+epsilon}}
$$

for all integers $p, q$ and $q$ is sufficiently large, and $mu=7.60631$, $epsilon>0$. This implies
$$
hleftlangle frac h{pi} rightrangle geq h^{2-mu-epsilon}
$$

for sufficiently large $h$. Then for some absolute constant $C>0$,
$$
left|frac1N sum_{nleq N} f(x_n) - frac1{2pi} int_{-pi}^{pi} f(x)dx right|leq C left( frac1m+ frac1N m^{mu-1+epsilon}right).
$$

Taking $m=lfloor N^{1/mu}rfloor$, we obtain
$$
left|frac1N sum_{nleq N} f(x_n) - frac1{2pi} int_{-pi}^{pi} f(x)dx right|leq C N^{-frac1{mu}+epsilon}.
$$

It is easy to see that $int_{-pi}^{pi} f(x) dx = 0$. Therefore,
$$
left|sum_{nleq N}f(x_n)right|leq CN^{1-frac1{mu}+epsilon}.
$$

The convergence of the series now follows from Abel's summation formula.



Added on 12/1/2018



By the bound for the discrepancy here: https://en.wikipedia.org/wiki/Low-discrepancy_sequence#Additive_recurrence



We may have a better error term:
$$
left|sum_{nleq N}f(x_n)right|leq CN^{1-frac1{mu-1}+epsilon}.
$$






share|cite|improve this answer











$endgroup$









  • 5




    $begingroup$
    Wow. You need a lot of sledgehammers to crack this nut.
    $endgroup$
    – marty cohen
    Mar 30 '17 at 0:39






  • 1




    $begingroup$
    @martycohen I will be interested to see if we can avoid them. But, I do not know any easier ways by now.
    $endgroup$
    – i707107
    Mar 30 '17 at 0:54






  • 1




    $begingroup$
    @i707107 Your solution is absolutely brilliant.
    $endgroup$
    – Sanjoy The Manjoy
    Mar 31 '17 at 4:36










  • $begingroup$
    Yes, we can avoid using the bound on the irrationality measure of $pi$ by splitting the sum in Erdős–Turán into the sum over the numbers $h$ that are denominators of the convergents of $1/pi$ and the sum over the rest.
    $endgroup$
    – Jarek Kuben
    May 10 '17 at 21:32












  • $begingroup$
    @JarekKuben If we really can avoid using irrationality measure of $pi$, please post your entire solution separately as an answer.
    $endgroup$
    – i707107
    May 11 '17 at 0:24



















2












$begingroup$

$color{red}{text{Not an answer, just an idea needing more work}}$ :





I'd say $sum_n frac{(-1)^n}{n} b_n$ converges whenever $Delta^k b(n) = O(w^{k}),w< 2$ where $Delta^k b(n)$ is the $k$th forward difference, here $b(n) = |sin n|$



If you sum by parts $k$ times, using that $sum_{n=1}^N frac{(-1)^n}{n} = frac{(-1)^N}{2 N}+ O(frac{1}{2 N^2})$ you'll get a main term $2^{-k} sum_{n=1}^{N-k} frac{(-1)^n}{n} Delta^k b(n)$





use that $A(2M)=sum_{n=M}^infty frac{1}{(2n+1)(2n+2)} = sum_{n=m}^infty frac{1}{(2n+1)^2}-frac{1}{(2n+1)^2(2n+2)}$ and approximate with $int_{2M}^infty frac{dx}{x^2}-int_{2M}^infty frac{dx}{x^3}$ to obtain



$$A(N) = ln 2+sum_{n=1}^N frac{(-1)^n}{n}=frac{(-1)^N}{2N}+O(frac{1}{2 N^2})$$



Summing by parts
$$sum_{n=1}^N frac{(-1)^n}{n} |sin n| = A(N)|sin(N)|+sum_{n=1}^{N-1} A(n) (|sin n| - |sin (n +1)|)$$



The problematic term is $sum_{n=1}^{N-1} frac{(-1)^N}{2N} (|sin n| - |sin (n +1)|) $
that we can sum by parts again to get a new problematic term
$$sum_{n=1}^{N-2} frac{(-1)^n}{4n} Delta^2 b(n)$$
where $Delta^2 b(n)=(|sin n| - |sin( n +1)|)-(|sin( n+1)| - |sin( n +2)|)$



summing by parts $k$ times we'll have
$$frac{1}{2^k}sum_{n=1}^{N-k} frac{(-1)^n}{n} Delta^k b(n)$$
Where $Delta^k b(n)$ is the $k$th forward difference of $b(n) = |sin n|$






share|cite|improve this answer











$endgroup$





















    0












    $begingroup$

    By using the Fourier series of $left|sin(n)right|$ the problem of showing that
    $$ sum_{ngeq 1}frac{(-1)^n left|sin nright|}{n} $$
    is convergent boils down to the problem of showing that
    $$ sum_{mgeq 1}frac{logleft|cos mright|}{4m^2-1}$$
    is convergent. The only issue is given by the values of $m$ such that $m$ is close to an odd multiple of $frac{pi}{2}$.

    On the other hand, since the irrationality measure of $pi$ is finite, even if $cos m$ is close to zero it cannot be closer than $frac{1}{m^{10}}$ for any $m$ large enough. Since
    $$ sum_{mgeq 1}frac{10log m}{4m^2-1} $$
    is absolutely convergent, the original series is convergent as well.






    share|cite|improve this answer









    $endgroup$














      Your Answer








      StackExchange.ready(function() {
      var channelOptions = {
      tags: "".split(" "),
      id: "69"
      };
      initTagRenderer("".split(" "), "".split(" "), channelOptions);

      StackExchange.using("externalEditor", function() {
      // Have to fire editor after snippets, if snippets enabled
      if (StackExchange.settings.snippets.snippetsEnabled) {
      StackExchange.using("snippets", function() {
      createEditor();
      });
      }
      else {
      createEditor();
      }
      });

      function createEditor() {
      StackExchange.prepareEditor({
      heartbeatType: 'answer',
      autoActivateHeartbeat: false,
      convertImagesToLinks: true,
      noModals: true,
      showLowRepImageUploadWarning: true,
      reputationToPostImages: 10,
      bindNavPrevention: true,
      postfix: "",
      imageUploader: {
      brandingHtml: "Powered by u003ca class="icon-imgur-white" href="https://imgur.com/"u003eu003c/au003e",
      contentPolicyHtml: "User contributions licensed under u003ca href="https://creativecommons.org/licenses/by-sa/3.0/"u003ecc by-sa 3.0 with attribution requiredu003c/au003e u003ca href="https://stackoverflow.com/legal/content-policy"u003e(content policy)u003c/au003e",
      allowUrls: true
      },
      noCode: true, onDemand: true,
      discardSelector: ".discard-answer"
      ,immediatelyShowMarkdownHelp:true
      });


      }
      });














      draft saved

      draft discarded


















      StackExchange.ready(
      function () {
      StackExchange.openid.initPostLogin('.new-post-login', 'https%3a%2f%2fmath.stackexchange.com%2fquestions%2f735283%2fconvergence-of-an-alternating-series-sum-n-geq-1-frac-1n-sin-nn%23new-answer', 'question_page');
      }
      );

      Post as a guest















      Required, but never shown
























      4 Answers
      4






      active

      oldest

      votes








      4 Answers
      4






      active

      oldest

      votes









      active

      oldest

      votes






      active

      oldest

      votes









      11





      +50







      $begingroup$

      By Dirichlet's convergence test, this series will converge if we can show that there exists a constant $C$ such that $$left|sum_{nleq x}(-1)^{n}|sin(n)|right|leq C$$ for all $x$.



      Lets write $$sum_{nleq x}(-1)^{n}|sin(n)|=sum_{nleqfrac{x}{2}}|sin(2n)|-sum_{nleqfrac{x+1}{2}}|sin(2n-1)|.$$ Then for $x=2N$, an even number, Euler Maclaurin summation yields $$sum_{nleq N}|sin(2n)|=int_{1}^{N}|sin(2t)|dt+sum_{k=1}^{K}frac{(-1)^{k}}{k!}B_{k}left(frac{d^{k-1}}{dt^{k-1}}|sin(2t)|biggr|_{t=1}^{t=N}right)$$ $$ -frac{(-1)^{K}}{K!}int_{1}^{N}B_{K}({t})left(frac{d^{k}}{dt^{k}}|sin(2t)|right)dt.$$ Note that $|sin(x)|$ has infinitely many derivatives everywhere except at integer multiples of $pi$, and so the above holds for any $K>0$. Since $$|B_{k}({x})|leq k!2^{1-k}pi^{-k}zeta(k),$$ and since the derivatives of $|sin(t)|$ are bounded in absolute value by $1$, it follows that $$left|sum_{nleq N}|sin(2n)|-int_{1}^{N}|sin(2t)|dtright|leq4sum_{k=1}^{K}frac{zeta(k)}{(2pi)^{k}}+frac{2zeta(K)N}{(2pi)^{K}}.$$ The series $sum_{k=1}^{infty}frac{zeta(k)}{(2pi)^{k}}$ converges absolutely, so by taking $K=N$ we see that there exists a constant $C_{1}$ such that $$left|sum_{nleq N}|sin(2n)|-int_{1}^{N}|sin(2t)|dtright|leq C_{1}$$ for all $N$. Similarly, there exists a constant $C_{2}$ such that $$left|sum_{nleq N}|sin(2n-1)|-int_{1}^{N}|sin(2t-1)|dtright|leq C_{2}.$$ Thus by the triangle inequality, $$left|sum_{nleq x}(-1)^{n}|sin(n)|right|leq C_{1}+C_{2}+left|int_{1}^{N}|sin(2t)|dt-int_{1}^{N}|sin(2t-1)|dtright|$$



      $$leq C_{1}+C_{2}+int_{N-1/2}^{N}|sin(2t)|dt+int_{1}^{3/2}|sin(2t-1)|dt$$



      $$leq C_{3}$$ for some constant $C_{3}$. This implies the desired result.






      share|cite|improve this answer









      $endgroup$









      • 2




        $begingroup$
        Euler Maclaurin summation formula might not work for functions with discontinuous derivatives.
        $endgroup$
        – i707107
        Apr 25 '17 at 1:23










      • $begingroup$
        See my answer to the same problem: math.stackexchange.com/questions/2202138/… This uses different idea, but it is quite a standard approach for such series.
        $endgroup$
        – i707107
        Apr 25 '17 at 1:31












      • $begingroup$
        See en.wikipedia.org/wiki/Euler%E2%80%93Maclaurin_formula for the notations I use, and for the case $p=1$, $S=sum_{m<jleq n} f(j)$, $I=int_m^n f(x) dx$, $$ S-I=frac{B_1}{1!}(f(n)-f(m)) + R_1$$ $$=frac{f(n)-f(m)}2 +int_m^n left( {x}-frac12right) df(x)$$ This is integration by parts for Riemann-Stiltjes integral. A bound for the remainder $$ |R_1|leq frac 2{2pi} int_m^n |f'(x)| dx,$$ in case $f'$ is continuous. But, it is $$ |R_1|leq frac 2{2pi} int_m^n |df(x)| $$ if $f'$ has discontinuity. The function $f(t)=|sin t|$ has a discontinuous derivative.
        $endgroup$
        – i707107
        Jan 9 at 0:06


















      11





      +50







      $begingroup$

      By Dirichlet's convergence test, this series will converge if we can show that there exists a constant $C$ such that $$left|sum_{nleq x}(-1)^{n}|sin(n)|right|leq C$$ for all $x$.



      Lets write $$sum_{nleq x}(-1)^{n}|sin(n)|=sum_{nleqfrac{x}{2}}|sin(2n)|-sum_{nleqfrac{x+1}{2}}|sin(2n-1)|.$$ Then for $x=2N$, an even number, Euler Maclaurin summation yields $$sum_{nleq N}|sin(2n)|=int_{1}^{N}|sin(2t)|dt+sum_{k=1}^{K}frac{(-1)^{k}}{k!}B_{k}left(frac{d^{k-1}}{dt^{k-1}}|sin(2t)|biggr|_{t=1}^{t=N}right)$$ $$ -frac{(-1)^{K}}{K!}int_{1}^{N}B_{K}({t})left(frac{d^{k}}{dt^{k}}|sin(2t)|right)dt.$$ Note that $|sin(x)|$ has infinitely many derivatives everywhere except at integer multiples of $pi$, and so the above holds for any $K>0$. Since $$|B_{k}({x})|leq k!2^{1-k}pi^{-k}zeta(k),$$ and since the derivatives of $|sin(t)|$ are bounded in absolute value by $1$, it follows that $$left|sum_{nleq N}|sin(2n)|-int_{1}^{N}|sin(2t)|dtright|leq4sum_{k=1}^{K}frac{zeta(k)}{(2pi)^{k}}+frac{2zeta(K)N}{(2pi)^{K}}.$$ The series $sum_{k=1}^{infty}frac{zeta(k)}{(2pi)^{k}}$ converges absolutely, so by taking $K=N$ we see that there exists a constant $C_{1}$ such that $$left|sum_{nleq N}|sin(2n)|-int_{1}^{N}|sin(2t)|dtright|leq C_{1}$$ for all $N$. Similarly, there exists a constant $C_{2}$ such that $$left|sum_{nleq N}|sin(2n-1)|-int_{1}^{N}|sin(2t-1)|dtright|leq C_{2}.$$ Thus by the triangle inequality, $$left|sum_{nleq x}(-1)^{n}|sin(n)|right|leq C_{1}+C_{2}+left|int_{1}^{N}|sin(2t)|dt-int_{1}^{N}|sin(2t-1)|dtright|$$



      $$leq C_{1}+C_{2}+int_{N-1/2}^{N}|sin(2t)|dt+int_{1}^{3/2}|sin(2t-1)|dt$$



      $$leq C_{3}$$ for some constant $C_{3}$. This implies the desired result.






      share|cite|improve this answer









      $endgroup$









      • 2




        $begingroup$
        Euler Maclaurin summation formula might not work for functions with discontinuous derivatives.
        $endgroup$
        – i707107
        Apr 25 '17 at 1:23










      • $begingroup$
        See my answer to the same problem: math.stackexchange.com/questions/2202138/… This uses different idea, but it is quite a standard approach for such series.
        $endgroup$
        – i707107
        Apr 25 '17 at 1:31












      • $begingroup$
        See en.wikipedia.org/wiki/Euler%E2%80%93Maclaurin_formula for the notations I use, and for the case $p=1$, $S=sum_{m<jleq n} f(j)$, $I=int_m^n f(x) dx$, $$ S-I=frac{B_1}{1!}(f(n)-f(m)) + R_1$$ $$=frac{f(n)-f(m)}2 +int_m^n left( {x}-frac12right) df(x)$$ This is integration by parts for Riemann-Stiltjes integral. A bound for the remainder $$ |R_1|leq frac 2{2pi} int_m^n |f'(x)| dx,$$ in case $f'$ is continuous. But, it is $$ |R_1|leq frac 2{2pi} int_m^n |df(x)| $$ if $f'$ has discontinuity. The function $f(t)=|sin t|$ has a discontinuous derivative.
        $endgroup$
        – i707107
        Jan 9 at 0:06
















      11





      +50







      11





      +50



      11




      +50



      $begingroup$

      By Dirichlet's convergence test, this series will converge if we can show that there exists a constant $C$ such that $$left|sum_{nleq x}(-1)^{n}|sin(n)|right|leq C$$ for all $x$.



      Lets write $$sum_{nleq x}(-1)^{n}|sin(n)|=sum_{nleqfrac{x}{2}}|sin(2n)|-sum_{nleqfrac{x+1}{2}}|sin(2n-1)|.$$ Then for $x=2N$, an even number, Euler Maclaurin summation yields $$sum_{nleq N}|sin(2n)|=int_{1}^{N}|sin(2t)|dt+sum_{k=1}^{K}frac{(-1)^{k}}{k!}B_{k}left(frac{d^{k-1}}{dt^{k-1}}|sin(2t)|biggr|_{t=1}^{t=N}right)$$ $$ -frac{(-1)^{K}}{K!}int_{1}^{N}B_{K}({t})left(frac{d^{k}}{dt^{k}}|sin(2t)|right)dt.$$ Note that $|sin(x)|$ has infinitely many derivatives everywhere except at integer multiples of $pi$, and so the above holds for any $K>0$. Since $$|B_{k}({x})|leq k!2^{1-k}pi^{-k}zeta(k),$$ and since the derivatives of $|sin(t)|$ are bounded in absolute value by $1$, it follows that $$left|sum_{nleq N}|sin(2n)|-int_{1}^{N}|sin(2t)|dtright|leq4sum_{k=1}^{K}frac{zeta(k)}{(2pi)^{k}}+frac{2zeta(K)N}{(2pi)^{K}}.$$ The series $sum_{k=1}^{infty}frac{zeta(k)}{(2pi)^{k}}$ converges absolutely, so by taking $K=N$ we see that there exists a constant $C_{1}$ such that $$left|sum_{nleq N}|sin(2n)|-int_{1}^{N}|sin(2t)|dtright|leq C_{1}$$ for all $N$. Similarly, there exists a constant $C_{2}$ such that $$left|sum_{nleq N}|sin(2n-1)|-int_{1}^{N}|sin(2t-1)|dtright|leq C_{2}.$$ Thus by the triangle inequality, $$left|sum_{nleq x}(-1)^{n}|sin(n)|right|leq C_{1}+C_{2}+left|int_{1}^{N}|sin(2t)|dt-int_{1}^{N}|sin(2t-1)|dtright|$$



      $$leq C_{1}+C_{2}+int_{N-1/2}^{N}|sin(2t)|dt+int_{1}^{3/2}|sin(2t-1)|dt$$



      $$leq C_{3}$$ for some constant $C_{3}$. This implies the desired result.






      share|cite|improve this answer









      $endgroup$



      By Dirichlet's convergence test, this series will converge if we can show that there exists a constant $C$ such that $$left|sum_{nleq x}(-1)^{n}|sin(n)|right|leq C$$ for all $x$.



      Lets write $$sum_{nleq x}(-1)^{n}|sin(n)|=sum_{nleqfrac{x}{2}}|sin(2n)|-sum_{nleqfrac{x+1}{2}}|sin(2n-1)|.$$ Then for $x=2N$, an even number, Euler Maclaurin summation yields $$sum_{nleq N}|sin(2n)|=int_{1}^{N}|sin(2t)|dt+sum_{k=1}^{K}frac{(-1)^{k}}{k!}B_{k}left(frac{d^{k-1}}{dt^{k-1}}|sin(2t)|biggr|_{t=1}^{t=N}right)$$ $$ -frac{(-1)^{K}}{K!}int_{1}^{N}B_{K}({t})left(frac{d^{k}}{dt^{k}}|sin(2t)|right)dt.$$ Note that $|sin(x)|$ has infinitely many derivatives everywhere except at integer multiples of $pi$, and so the above holds for any $K>0$. Since $$|B_{k}({x})|leq k!2^{1-k}pi^{-k}zeta(k),$$ and since the derivatives of $|sin(t)|$ are bounded in absolute value by $1$, it follows that $$left|sum_{nleq N}|sin(2n)|-int_{1}^{N}|sin(2t)|dtright|leq4sum_{k=1}^{K}frac{zeta(k)}{(2pi)^{k}}+frac{2zeta(K)N}{(2pi)^{K}}.$$ The series $sum_{k=1}^{infty}frac{zeta(k)}{(2pi)^{k}}$ converges absolutely, so by taking $K=N$ we see that there exists a constant $C_{1}$ such that $$left|sum_{nleq N}|sin(2n)|-int_{1}^{N}|sin(2t)|dtright|leq C_{1}$$ for all $N$. Similarly, there exists a constant $C_{2}$ such that $$left|sum_{nleq N}|sin(2n-1)|-int_{1}^{N}|sin(2t-1)|dtright|leq C_{2}.$$ Thus by the triangle inequality, $$left|sum_{nleq x}(-1)^{n}|sin(n)|right|leq C_{1}+C_{2}+left|int_{1}^{N}|sin(2t)|dt-int_{1}^{N}|sin(2t-1)|dtright|$$



      $$leq C_{1}+C_{2}+int_{N-1/2}^{N}|sin(2t)|dt+int_{1}^{3/2}|sin(2t-1)|dt$$



      $$leq C_{3}$$ for some constant $C_{3}$. This implies the desired result.







      share|cite|improve this answer












      share|cite|improve this answer



      share|cite|improve this answer










      answered Apr 7 '14 at 1:28









      Eric NaslundEric Naslund

      60.7k10140242




      60.7k10140242








      • 2




        $begingroup$
        Euler Maclaurin summation formula might not work for functions with discontinuous derivatives.
        $endgroup$
        – i707107
        Apr 25 '17 at 1:23










      • $begingroup$
        See my answer to the same problem: math.stackexchange.com/questions/2202138/… This uses different idea, but it is quite a standard approach for such series.
        $endgroup$
        – i707107
        Apr 25 '17 at 1:31












      • $begingroup$
        See en.wikipedia.org/wiki/Euler%E2%80%93Maclaurin_formula for the notations I use, and for the case $p=1$, $S=sum_{m<jleq n} f(j)$, $I=int_m^n f(x) dx$, $$ S-I=frac{B_1}{1!}(f(n)-f(m)) + R_1$$ $$=frac{f(n)-f(m)}2 +int_m^n left( {x}-frac12right) df(x)$$ This is integration by parts for Riemann-Stiltjes integral. A bound for the remainder $$ |R_1|leq frac 2{2pi} int_m^n |f'(x)| dx,$$ in case $f'$ is continuous. But, it is $$ |R_1|leq frac 2{2pi} int_m^n |df(x)| $$ if $f'$ has discontinuity. The function $f(t)=|sin t|$ has a discontinuous derivative.
        $endgroup$
        – i707107
        Jan 9 at 0:06
















      • 2




        $begingroup$
        Euler Maclaurin summation formula might not work for functions with discontinuous derivatives.
        $endgroup$
        – i707107
        Apr 25 '17 at 1:23










      • $begingroup$
        See my answer to the same problem: math.stackexchange.com/questions/2202138/… This uses different idea, but it is quite a standard approach for such series.
        $endgroup$
        – i707107
        Apr 25 '17 at 1:31












      • $begingroup$
        See en.wikipedia.org/wiki/Euler%E2%80%93Maclaurin_formula for the notations I use, and for the case $p=1$, $S=sum_{m<jleq n} f(j)$, $I=int_m^n f(x) dx$, $$ S-I=frac{B_1}{1!}(f(n)-f(m)) + R_1$$ $$=frac{f(n)-f(m)}2 +int_m^n left( {x}-frac12right) df(x)$$ This is integration by parts for Riemann-Stiltjes integral. A bound for the remainder $$ |R_1|leq frac 2{2pi} int_m^n |f'(x)| dx,$$ in case $f'$ is continuous. But, it is $$ |R_1|leq frac 2{2pi} int_m^n |df(x)| $$ if $f'$ has discontinuity. The function $f(t)=|sin t|$ has a discontinuous derivative.
        $endgroup$
        – i707107
        Jan 9 at 0:06










      2




      2




      $begingroup$
      Euler Maclaurin summation formula might not work for functions with discontinuous derivatives.
      $endgroup$
      – i707107
      Apr 25 '17 at 1:23




      $begingroup$
      Euler Maclaurin summation formula might not work for functions with discontinuous derivatives.
      $endgroup$
      – i707107
      Apr 25 '17 at 1:23












      $begingroup$
      See my answer to the same problem: math.stackexchange.com/questions/2202138/… This uses different idea, but it is quite a standard approach for such series.
      $endgroup$
      – i707107
      Apr 25 '17 at 1:31






      $begingroup$
      See my answer to the same problem: math.stackexchange.com/questions/2202138/… This uses different idea, but it is quite a standard approach for such series.
      $endgroup$
      – i707107
      Apr 25 '17 at 1:31














      $begingroup$
      See en.wikipedia.org/wiki/Euler%E2%80%93Maclaurin_formula for the notations I use, and for the case $p=1$, $S=sum_{m<jleq n} f(j)$, $I=int_m^n f(x) dx$, $$ S-I=frac{B_1}{1!}(f(n)-f(m)) + R_1$$ $$=frac{f(n)-f(m)}2 +int_m^n left( {x}-frac12right) df(x)$$ This is integration by parts for Riemann-Stiltjes integral. A bound for the remainder $$ |R_1|leq frac 2{2pi} int_m^n |f'(x)| dx,$$ in case $f'$ is continuous. But, it is $$ |R_1|leq frac 2{2pi} int_m^n |df(x)| $$ if $f'$ has discontinuity. The function $f(t)=|sin t|$ has a discontinuous derivative.
      $endgroup$
      – i707107
      Jan 9 at 0:06






      $begingroup$
      See en.wikipedia.org/wiki/Euler%E2%80%93Maclaurin_formula for the notations I use, and for the case $p=1$, $S=sum_{m<jleq n} f(j)$, $I=int_m^n f(x) dx$, $$ S-I=frac{B_1}{1!}(f(n)-f(m)) + R_1$$ $$=frac{f(n)-f(m)}2 +int_m^n left( {x}-frac12right) df(x)$$ This is integration by parts for Riemann-Stiltjes integral. A bound for the remainder $$ |R_1|leq frac 2{2pi} int_m^n |f'(x)| dx,$$ in case $f'$ is continuous. But, it is $$ |R_1|leq frac 2{2pi} int_m^n |df(x)| $$ if $f'$ has discontinuity. The function $f(t)=|sin t|$ has a discontinuous derivative.
      $endgroup$
      – i707107
      Jan 9 at 0:06













      12












      $begingroup$

      The series converges. It is enough to show that the sequence of the following partial sums converges:
      begin{align}
      s_N &= sum_{n=1}^{N} left(frac{ |sin 2n|}{2n}-frac{|sin (2n+1)|}{2n+1}right)=sum_{n=1}^N left(frac{(2n+1)|sin 2n|- 2n|sin (2n+1)| }{2n(2n+1)} right)\
      &=sum_{n=1}^N left( frac{|sin 2n|-|sin (2n+1)|}{2n+1}+frac{|sin 2n|}{2n(2n+1)}right).
      end{align}

      Thus, it is enough to show that the following converges:
      $$
      S_N = sum_{n=1}^N frac{|sin 2n|-|sin (2n+1)|}{2n+1}.
      $$

      We consider a partition of $mathbb{N}$ into four disjoint sets $A_{1}$, $A_{2}$, $A_{3}$, $A_{4}$ defined by:
      $$
      A_{1}={ninmathbb{N}: sin 2n >0, sin (2n+1)>0}, A_{2}={ninmathbb{N}: sin 2n >0, sin (2n+1)<0},
      $$

      $$
      A_{3}={ninmathbb{N}: sin 2n <0, sin (2n+1)>0}, A_{4}={ninmathbb{N}: sin 2n <0, sin (2n+1)<0}.
      $$

      Note that
      $$
      A_{1}={ninmathbb{N}: 2n mathrm{mod} 2pi in (0,pi-1)}, A_{2}={ninmathbb{N}: 2n mathrm{mod} 2pi in (pi-1,pi)},$$

      $$
      A_{3}={ninmathbb{N}: 2n mathrm{mod} 2pi in (-1,0)}, A_{4}={ninmathbb{N}: 2n mathrm{mod} 2pi in (-pi, -1)}.$$

      By trigonometric identities,
      $$
      nin A_{1} Longrightarrow |sin 2n|-|sin (2n+1)| = sin 2n - sin(2n+1) = -2cos(2n+frac12)sin frac12, $$

      $$
      nin A_{2} Longrightarrow |sin 2n|-|sin (2n+1)| = sin 2n + sin(2n+1) = 2sin(2n+frac12)cos frac12, $$

      $$
      nin A_{3} Longrightarrow |sin 2n|-|sin (2n+1)| = -sin 2n - sin(2n+1) = -2sin(2n+frac12)cosfrac12, $$

      $$
      nin A_{4} Longrightarrow |sin 2n|-|sin (2n+1)| = -sin 2n + sin(2n+1) = 2cos(2n+frac12)sin frac12.$$

      We define
      $$
      f_1(x)=-I_{(0,pi-1)}(x)2cos(x+frac12)sinfrac12, f_2(x)=I_{(pi-1,pi)}(x)2sin(x+frac12)cosfrac12,$$

      $$
      f_3(x)=-I_{(-1,0)}(x)2sin(x+frac12)cosfrac12, f_4(x)=I_{(-pi,-1)}(x)2cos(x+frac12)sinfrac12 $$

      where $I_A$ is the characteristic function of $A$. Note that these functions $f_i(x)$ are of bounded variation in $[-pi,pi]$. Thus, $f=f_1+f_2+f_3+f_4$ is of a bounded variation.



      We need Koksma's inequality p. 143, Theorem 5.1 of 'Uniform Distribution of Sequences' by Kuipers and Niederreiter:




      Theorem [Koksma]



      Let $f$ be a function on $I=[0,1]$ of bounded variation $V(f)$, and suppose we are given $N$ points $x_1, ldots , x_N$ in $I$ with discrepancy
      $$
      D_N:=sup_{0leq aleq bleq 1} left|frac1N #{1leq nleq N: x_n in (a,b) } -(b-a)right|.
      $$

      Then
      $$
      left|frac1N sum_{nleq N} f(x_n) - int_I f(x)dx right|leq V(f)D_N.
      $$




      To control the discrepancy, we apply Erdos-Turan inequality p. 112, Theorem 2.5 of Kuipers and Niederreiter's book.




      Theorem[Erdos-Turan]



      Let $x_1, ldots, x_N$ be $N$ points in $I=[0,1]$. Then there is an absolute constant $C>0$ such that for any positive integer $m$,
      $$
      D_Nleq C left( frac1m+ sum_{h=1}^m frac1h left| frac1Nsum_{n=1}^N e^{2pi i h x_n}right|right).
      $$




      The sequence of our interest is $x_n = 2n$ mod $2pi$. The above two inequalities together applied to $f=f_1+f_2+f_3+f_4$ yields the following: There is an absolute constant $C>0$ such that for any positive integer $m$,
      $$
      left|frac1N sum_{nleq N} f(x_n) - frac1{2pi} int_{-pi}^{pi} f(x)dx right|leq C left( frac1m+ frac1Nsum_{h=1}^m frac1{hlangle frac h{pi} rangle}right).
      $$

      A result on the irrationality measure of $pi$ by Salikhov implies that
      $$
      left| frac1{pi} - frac pq right| geq frac 1{q^{mu+epsilon}}
      $$

      for all integers $p, q$ and $q$ is sufficiently large, and $mu=7.60631$, $epsilon>0$. This implies
      $$
      hleftlangle frac h{pi} rightrangle geq h^{2-mu-epsilon}
      $$

      for sufficiently large $h$. Then for some absolute constant $C>0$,
      $$
      left|frac1N sum_{nleq N} f(x_n) - frac1{2pi} int_{-pi}^{pi} f(x)dx right|leq C left( frac1m+ frac1N m^{mu-1+epsilon}right).
      $$

      Taking $m=lfloor N^{1/mu}rfloor$, we obtain
      $$
      left|frac1N sum_{nleq N} f(x_n) - frac1{2pi} int_{-pi}^{pi} f(x)dx right|leq C N^{-frac1{mu}+epsilon}.
      $$

      It is easy to see that $int_{-pi}^{pi} f(x) dx = 0$. Therefore,
      $$
      left|sum_{nleq N}f(x_n)right|leq CN^{1-frac1{mu}+epsilon}.
      $$

      The convergence of the series now follows from Abel's summation formula.



      Added on 12/1/2018



      By the bound for the discrepancy here: https://en.wikipedia.org/wiki/Low-discrepancy_sequence#Additive_recurrence



      We may have a better error term:
      $$
      left|sum_{nleq N}f(x_n)right|leq CN^{1-frac1{mu-1}+epsilon}.
      $$






      share|cite|improve this answer











      $endgroup$









      • 5




        $begingroup$
        Wow. You need a lot of sledgehammers to crack this nut.
        $endgroup$
        – marty cohen
        Mar 30 '17 at 0:39






      • 1




        $begingroup$
        @martycohen I will be interested to see if we can avoid them. But, I do not know any easier ways by now.
        $endgroup$
        – i707107
        Mar 30 '17 at 0:54






      • 1




        $begingroup$
        @i707107 Your solution is absolutely brilliant.
        $endgroup$
        – Sanjoy The Manjoy
        Mar 31 '17 at 4:36










      • $begingroup$
        Yes, we can avoid using the bound on the irrationality measure of $pi$ by splitting the sum in Erdős–Turán into the sum over the numbers $h$ that are denominators of the convergents of $1/pi$ and the sum over the rest.
        $endgroup$
        – Jarek Kuben
        May 10 '17 at 21:32












      • $begingroup$
        @JarekKuben If we really can avoid using irrationality measure of $pi$, please post your entire solution separately as an answer.
        $endgroup$
        – i707107
        May 11 '17 at 0:24
















      12












      $begingroup$

      The series converges. It is enough to show that the sequence of the following partial sums converges:
      begin{align}
      s_N &= sum_{n=1}^{N} left(frac{ |sin 2n|}{2n}-frac{|sin (2n+1)|}{2n+1}right)=sum_{n=1}^N left(frac{(2n+1)|sin 2n|- 2n|sin (2n+1)| }{2n(2n+1)} right)\
      &=sum_{n=1}^N left( frac{|sin 2n|-|sin (2n+1)|}{2n+1}+frac{|sin 2n|}{2n(2n+1)}right).
      end{align}

      Thus, it is enough to show that the following converges:
      $$
      S_N = sum_{n=1}^N frac{|sin 2n|-|sin (2n+1)|}{2n+1}.
      $$

      We consider a partition of $mathbb{N}$ into four disjoint sets $A_{1}$, $A_{2}$, $A_{3}$, $A_{4}$ defined by:
      $$
      A_{1}={ninmathbb{N}: sin 2n >0, sin (2n+1)>0}, A_{2}={ninmathbb{N}: sin 2n >0, sin (2n+1)<0},
      $$

      $$
      A_{3}={ninmathbb{N}: sin 2n <0, sin (2n+1)>0}, A_{4}={ninmathbb{N}: sin 2n <0, sin (2n+1)<0}.
      $$

      Note that
      $$
      A_{1}={ninmathbb{N}: 2n mathrm{mod} 2pi in (0,pi-1)}, A_{2}={ninmathbb{N}: 2n mathrm{mod} 2pi in (pi-1,pi)},$$

      $$
      A_{3}={ninmathbb{N}: 2n mathrm{mod} 2pi in (-1,0)}, A_{4}={ninmathbb{N}: 2n mathrm{mod} 2pi in (-pi, -1)}.$$

      By trigonometric identities,
      $$
      nin A_{1} Longrightarrow |sin 2n|-|sin (2n+1)| = sin 2n - sin(2n+1) = -2cos(2n+frac12)sin frac12, $$

      $$
      nin A_{2} Longrightarrow |sin 2n|-|sin (2n+1)| = sin 2n + sin(2n+1) = 2sin(2n+frac12)cos frac12, $$

      $$
      nin A_{3} Longrightarrow |sin 2n|-|sin (2n+1)| = -sin 2n - sin(2n+1) = -2sin(2n+frac12)cosfrac12, $$

      $$
      nin A_{4} Longrightarrow |sin 2n|-|sin (2n+1)| = -sin 2n + sin(2n+1) = 2cos(2n+frac12)sin frac12.$$

      We define
      $$
      f_1(x)=-I_{(0,pi-1)}(x)2cos(x+frac12)sinfrac12, f_2(x)=I_{(pi-1,pi)}(x)2sin(x+frac12)cosfrac12,$$

      $$
      f_3(x)=-I_{(-1,0)}(x)2sin(x+frac12)cosfrac12, f_4(x)=I_{(-pi,-1)}(x)2cos(x+frac12)sinfrac12 $$

      where $I_A$ is the characteristic function of $A$. Note that these functions $f_i(x)$ are of bounded variation in $[-pi,pi]$. Thus, $f=f_1+f_2+f_3+f_4$ is of a bounded variation.



      We need Koksma's inequality p. 143, Theorem 5.1 of 'Uniform Distribution of Sequences' by Kuipers and Niederreiter:




      Theorem [Koksma]



      Let $f$ be a function on $I=[0,1]$ of bounded variation $V(f)$, and suppose we are given $N$ points $x_1, ldots , x_N$ in $I$ with discrepancy
      $$
      D_N:=sup_{0leq aleq bleq 1} left|frac1N #{1leq nleq N: x_n in (a,b) } -(b-a)right|.
      $$

      Then
      $$
      left|frac1N sum_{nleq N} f(x_n) - int_I f(x)dx right|leq V(f)D_N.
      $$




      To control the discrepancy, we apply Erdos-Turan inequality p. 112, Theorem 2.5 of Kuipers and Niederreiter's book.




      Theorem[Erdos-Turan]



      Let $x_1, ldots, x_N$ be $N$ points in $I=[0,1]$. Then there is an absolute constant $C>0$ such that for any positive integer $m$,
      $$
      D_Nleq C left( frac1m+ sum_{h=1}^m frac1h left| frac1Nsum_{n=1}^N e^{2pi i h x_n}right|right).
      $$




      The sequence of our interest is $x_n = 2n$ mod $2pi$. The above two inequalities together applied to $f=f_1+f_2+f_3+f_4$ yields the following: There is an absolute constant $C>0$ such that for any positive integer $m$,
      $$
      left|frac1N sum_{nleq N} f(x_n) - frac1{2pi} int_{-pi}^{pi} f(x)dx right|leq C left( frac1m+ frac1Nsum_{h=1}^m frac1{hlangle frac h{pi} rangle}right).
      $$

      A result on the irrationality measure of $pi$ by Salikhov implies that
      $$
      left| frac1{pi} - frac pq right| geq frac 1{q^{mu+epsilon}}
      $$

      for all integers $p, q$ and $q$ is sufficiently large, and $mu=7.60631$, $epsilon>0$. This implies
      $$
      hleftlangle frac h{pi} rightrangle geq h^{2-mu-epsilon}
      $$

      for sufficiently large $h$. Then for some absolute constant $C>0$,
      $$
      left|frac1N sum_{nleq N} f(x_n) - frac1{2pi} int_{-pi}^{pi} f(x)dx right|leq C left( frac1m+ frac1N m^{mu-1+epsilon}right).
      $$

      Taking $m=lfloor N^{1/mu}rfloor$, we obtain
      $$
      left|frac1N sum_{nleq N} f(x_n) - frac1{2pi} int_{-pi}^{pi} f(x)dx right|leq C N^{-frac1{mu}+epsilon}.
      $$

      It is easy to see that $int_{-pi}^{pi} f(x) dx = 0$. Therefore,
      $$
      left|sum_{nleq N}f(x_n)right|leq CN^{1-frac1{mu}+epsilon}.
      $$

      The convergence of the series now follows from Abel's summation formula.



      Added on 12/1/2018



      By the bound for the discrepancy here: https://en.wikipedia.org/wiki/Low-discrepancy_sequence#Additive_recurrence



      We may have a better error term:
      $$
      left|sum_{nleq N}f(x_n)right|leq CN^{1-frac1{mu-1}+epsilon}.
      $$






      share|cite|improve this answer











      $endgroup$









      • 5




        $begingroup$
        Wow. You need a lot of sledgehammers to crack this nut.
        $endgroup$
        – marty cohen
        Mar 30 '17 at 0:39






      • 1




        $begingroup$
        @martycohen I will be interested to see if we can avoid them. But, I do not know any easier ways by now.
        $endgroup$
        – i707107
        Mar 30 '17 at 0:54






      • 1




        $begingroup$
        @i707107 Your solution is absolutely brilliant.
        $endgroup$
        – Sanjoy The Manjoy
        Mar 31 '17 at 4:36










      • $begingroup$
        Yes, we can avoid using the bound on the irrationality measure of $pi$ by splitting the sum in Erdős–Turán into the sum over the numbers $h$ that are denominators of the convergents of $1/pi$ and the sum over the rest.
        $endgroup$
        – Jarek Kuben
        May 10 '17 at 21:32












      • $begingroup$
        @JarekKuben If we really can avoid using irrationality measure of $pi$, please post your entire solution separately as an answer.
        $endgroup$
        – i707107
        May 11 '17 at 0:24














      12












      12








      12





      $begingroup$

      The series converges. It is enough to show that the sequence of the following partial sums converges:
      begin{align}
      s_N &= sum_{n=1}^{N} left(frac{ |sin 2n|}{2n}-frac{|sin (2n+1)|}{2n+1}right)=sum_{n=1}^N left(frac{(2n+1)|sin 2n|- 2n|sin (2n+1)| }{2n(2n+1)} right)\
      &=sum_{n=1}^N left( frac{|sin 2n|-|sin (2n+1)|}{2n+1}+frac{|sin 2n|}{2n(2n+1)}right).
      end{align}

      Thus, it is enough to show that the following converges:
      $$
      S_N = sum_{n=1}^N frac{|sin 2n|-|sin (2n+1)|}{2n+1}.
      $$

      We consider a partition of $mathbb{N}$ into four disjoint sets $A_{1}$, $A_{2}$, $A_{3}$, $A_{4}$ defined by:
      $$
      A_{1}={ninmathbb{N}: sin 2n >0, sin (2n+1)>0}, A_{2}={ninmathbb{N}: sin 2n >0, sin (2n+1)<0},
      $$

      $$
      A_{3}={ninmathbb{N}: sin 2n <0, sin (2n+1)>0}, A_{4}={ninmathbb{N}: sin 2n <0, sin (2n+1)<0}.
      $$

      Note that
      $$
      A_{1}={ninmathbb{N}: 2n mathrm{mod} 2pi in (0,pi-1)}, A_{2}={ninmathbb{N}: 2n mathrm{mod} 2pi in (pi-1,pi)},$$

      $$
      A_{3}={ninmathbb{N}: 2n mathrm{mod} 2pi in (-1,0)}, A_{4}={ninmathbb{N}: 2n mathrm{mod} 2pi in (-pi, -1)}.$$

      By trigonometric identities,
      $$
      nin A_{1} Longrightarrow |sin 2n|-|sin (2n+1)| = sin 2n - sin(2n+1) = -2cos(2n+frac12)sin frac12, $$

      $$
      nin A_{2} Longrightarrow |sin 2n|-|sin (2n+1)| = sin 2n + sin(2n+1) = 2sin(2n+frac12)cos frac12, $$

      $$
      nin A_{3} Longrightarrow |sin 2n|-|sin (2n+1)| = -sin 2n - sin(2n+1) = -2sin(2n+frac12)cosfrac12, $$

      $$
      nin A_{4} Longrightarrow |sin 2n|-|sin (2n+1)| = -sin 2n + sin(2n+1) = 2cos(2n+frac12)sin frac12.$$

      We define
      $$
      f_1(x)=-I_{(0,pi-1)}(x)2cos(x+frac12)sinfrac12, f_2(x)=I_{(pi-1,pi)}(x)2sin(x+frac12)cosfrac12,$$

      $$
      f_3(x)=-I_{(-1,0)}(x)2sin(x+frac12)cosfrac12, f_4(x)=I_{(-pi,-1)}(x)2cos(x+frac12)sinfrac12 $$

      where $I_A$ is the characteristic function of $A$. Note that these functions $f_i(x)$ are of bounded variation in $[-pi,pi]$. Thus, $f=f_1+f_2+f_3+f_4$ is of a bounded variation.



      We need Koksma's inequality p. 143, Theorem 5.1 of 'Uniform Distribution of Sequences' by Kuipers and Niederreiter:




      Theorem [Koksma]



      Let $f$ be a function on $I=[0,1]$ of bounded variation $V(f)$, and suppose we are given $N$ points $x_1, ldots , x_N$ in $I$ with discrepancy
      $$
      D_N:=sup_{0leq aleq bleq 1} left|frac1N #{1leq nleq N: x_n in (a,b) } -(b-a)right|.
      $$

      Then
      $$
      left|frac1N sum_{nleq N} f(x_n) - int_I f(x)dx right|leq V(f)D_N.
      $$




      To control the discrepancy, we apply Erdos-Turan inequality p. 112, Theorem 2.5 of Kuipers and Niederreiter's book.




      Theorem[Erdos-Turan]



      Let $x_1, ldots, x_N$ be $N$ points in $I=[0,1]$. Then there is an absolute constant $C>0$ such that for any positive integer $m$,
      $$
      D_Nleq C left( frac1m+ sum_{h=1}^m frac1h left| frac1Nsum_{n=1}^N e^{2pi i h x_n}right|right).
      $$




      The sequence of our interest is $x_n = 2n$ mod $2pi$. The above two inequalities together applied to $f=f_1+f_2+f_3+f_4$ yields the following: There is an absolute constant $C>0$ such that for any positive integer $m$,
      $$
      left|frac1N sum_{nleq N} f(x_n) - frac1{2pi} int_{-pi}^{pi} f(x)dx right|leq C left( frac1m+ frac1Nsum_{h=1}^m frac1{hlangle frac h{pi} rangle}right).
      $$

      A result on the irrationality measure of $pi$ by Salikhov implies that
      $$
      left| frac1{pi} - frac pq right| geq frac 1{q^{mu+epsilon}}
      $$

      for all integers $p, q$ and $q$ is sufficiently large, and $mu=7.60631$, $epsilon>0$. This implies
      $$
      hleftlangle frac h{pi} rightrangle geq h^{2-mu-epsilon}
      $$

      for sufficiently large $h$. Then for some absolute constant $C>0$,
      $$
      left|frac1N sum_{nleq N} f(x_n) - frac1{2pi} int_{-pi}^{pi} f(x)dx right|leq C left( frac1m+ frac1N m^{mu-1+epsilon}right).
      $$

      Taking $m=lfloor N^{1/mu}rfloor$, we obtain
      $$
      left|frac1N sum_{nleq N} f(x_n) - frac1{2pi} int_{-pi}^{pi} f(x)dx right|leq C N^{-frac1{mu}+epsilon}.
      $$

      It is easy to see that $int_{-pi}^{pi} f(x) dx = 0$. Therefore,
      $$
      left|sum_{nleq N}f(x_n)right|leq CN^{1-frac1{mu}+epsilon}.
      $$

      The convergence of the series now follows from Abel's summation formula.



      Added on 12/1/2018



      By the bound for the discrepancy here: https://en.wikipedia.org/wiki/Low-discrepancy_sequence#Additive_recurrence



      We may have a better error term:
      $$
      left|sum_{nleq N}f(x_n)right|leq CN^{1-frac1{mu-1}+epsilon}.
      $$






      share|cite|improve this answer











      $endgroup$



      The series converges. It is enough to show that the sequence of the following partial sums converges:
      begin{align}
      s_N &= sum_{n=1}^{N} left(frac{ |sin 2n|}{2n}-frac{|sin (2n+1)|}{2n+1}right)=sum_{n=1}^N left(frac{(2n+1)|sin 2n|- 2n|sin (2n+1)| }{2n(2n+1)} right)\
      &=sum_{n=1}^N left( frac{|sin 2n|-|sin (2n+1)|}{2n+1}+frac{|sin 2n|}{2n(2n+1)}right).
      end{align}

      Thus, it is enough to show that the following converges:
      $$
      S_N = sum_{n=1}^N frac{|sin 2n|-|sin (2n+1)|}{2n+1}.
      $$

      We consider a partition of $mathbb{N}$ into four disjoint sets $A_{1}$, $A_{2}$, $A_{3}$, $A_{4}$ defined by:
      $$
      A_{1}={ninmathbb{N}: sin 2n >0, sin (2n+1)>0}, A_{2}={ninmathbb{N}: sin 2n >0, sin (2n+1)<0},
      $$

      $$
      A_{3}={ninmathbb{N}: sin 2n <0, sin (2n+1)>0}, A_{4}={ninmathbb{N}: sin 2n <0, sin (2n+1)<0}.
      $$

      Note that
      $$
      A_{1}={ninmathbb{N}: 2n mathrm{mod} 2pi in (0,pi-1)}, A_{2}={ninmathbb{N}: 2n mathrm{mod} 2pi in (pi-1,pi)},$$

      $$
      A_{3}={ninmathbb{N}: 2n mathrm{mod} 2pi in (-1,0)}, A_{4}={ninmathbb{N}: 2n mathrm{mod} 2pi in (-pi, -1)}.$$

      By trigonometric identities,
      $$
      nin A_{1} Longrightarrow |sin 2n|-|sin (2n+1)| = sin 2n - sin(2n+1) = -2cos(2n+frac12)sin frac12, $$

      $$
      nin A_{2} Longrightarrow |sin 2n|-|sin (2n+1)| = sin 2n + sin(2n+1) = 2sin(2n+frac12)cos frac12, $$

      $$
      nin A_{3} Longrightarrow |sin 2n|-|sin (2n+1)| = -sin 2n - sin(2n+1) = -2sin(2n+frac12)cosfrac12, $$

      $$
      nin A_{4} Longrightarrow |sin 2n|-|sin (2n+1)| = -sin 2n + sin(2n+1) = 2cos(2n+frac12)sin frac12.$$

      We define
      $$
      f_1(x)=-I_{(0,pi-1)}(x)2cos(x+frac12)sinfrac12, f_2(x)=I_{(pi-1,pi)}(x)2sin(x+frac12)cosfrac12,$$

      $$
      f_3(x)=-I_{(-1,0)}(x)2sin(x+frac12)cosfrac12, f_4(x)=I_{(-pi,-1)}(x)2cos(x+frac12)sinfrac12 $$

      where $I_A$ is the characteristic function of $A$. Note that these functions $f_i(x)$ are of bounded variation in $[-pi,pi]$. Thus, $f=f_1+f_2+f_3+f_4$ is of a bounded variation.



      We need Koksma's inequality p. 143, Theorem 5.1 of 'Uniform Distribution of Sequences' by Kuipers and Niederreiter:




      Theorem [Koksma]



      Let $f$ be a function on $I=[0,1]$ of bounded variation $V(f)$, and suppose we are given $N$ points $x_1, ldots , x_N$ in $I$ with discrepancy
      $$
      D_N:=sup_{0leq aleq bleq 1} left|frac1N #{1leq nleq N: x_n in (a,b) } -(b-a)right|.
      $$

      Then
      $$
      left|frac1N sum_{nleq N} f(x_n) - int_I f(x)dx right|leq V(f)D_N.
      $$




      To control the discrepancy, we apply Erdos-Turan inequality p. 112, Theorem 2.5 of Kuipers and Niederreiter's book.




      Theorem[Erdos-Turan]



      Let $x_1, ldots, x_N$ be $N$ points in $I=[0,1]$. Then there is an absolute constant $C>0$ such that for any positive integer $m$,
      $$
      D_Nleq C left( frac1m+ sum_{h=1}^m frac1h left| frac1Nsum_{n=1}^N e^{2pi i h x_n}right|right).
      $$




      The sequence of our interest is $x_n = 2n$ mod $2pi$. The above two inequalities together applied to $f=f_1+f_2+f_3+f_4$ yields the following: There is an absolute constant $C>0$ such that for any positive integer $m$,
      $$
      left|frac1N sum_{nleq N} f(x_n) - frac1{2pi} int_{-pi}^{pi} f(x)dx right|leq C left( frac1m+ frac1Nsum_{h=1}^m frac1{hlangle frac h{pi} rangle}right).
      $$

      A result on the irrationality measure of $pi$ by Salikhov implies that
      $$
      left| frac1{pi} - frac pq right| geq frac 1{q^{mu+epsilon}}
      $$

      for all integers $p, q$ and $q$ is sufficiently large, and $mu=7.60631$, $epsilon>0$. This implies
      $$
      hleftlangle frac h{pi} rightrangle geq h^{2-mu-epsilon}
      $$

      for sufficiently large $h$. Then for some absolute constant $C>0$,
      $$
      left|frac1N sum_{nleq N} f(x_n) - frac1{2pi} int_{-pi}^{pi} f(x)dx right|leq C left( frac1m+ frac1N m^{mu-1+epsilon}right).
      $$

      Taking $m=lfloor N^{1/mu}rfloor$, we obtain
      $$
      left|frac1N sum_{nleq N} f(x_n) - frac1{2pi} int_{-pi}^{pi} f(x)dx right|leq C N^{-frac1{mu}+epsilon}.
      $$

      It is easy to see that $int_{-pi}^{pi} f(x) dx = 0$. Therefore,
      $$
      left|sum_{nleq N}f(x_n)right|leq CN^{1-frac1{mu}+epsilon}.
      $$

      The convergence of the series now follows from Abel's summation formula.



      Added on 12/1/2018



      By the bound for the discrepancy here: https://en.wikipedia.org/wiki/Low-discrepancy_sequence#Additive_recurrence



      We may have a better error term:
      $$
      left|sum_{nleq N}f(x_n)right|leq CN^{1-frac1{mu-1}+epsilon}.
      $$







      share|cite|improve this answer














      share|cite|improve this answer



      share|cite|improve this answer








      edited Dec 1 '18 at 14:19

























      answered Mar 28 '17 at 17:45









      i707107i707107

      12.7k21647




      12.7k21647








      • 5




        $begingroup$
        Wow. You need a lot of sledgehammers to crack this nut.
        $endgroup$
        – marty cohen
        Mar 30 '17 at 0:39






      • 1




        $begingroup$
        @martycohen I will be interested to see if we can avoid them. But, I do not know any easier ways by now.
        $endgroup$
        – i707107
        Mar 30 '17 at 0:54






      • 1




        $begingroup$
        @i707107 Your solution is absolutely brilliant.
        $endgroup$
        – Sanjoy The Manjoy
        Mar 31 '17 at 4:36










      • $begingroup$
        Yes, we can avoid using the bound on the irrationality measure of $pi$ by splitting the sum in Erdős–Turán into the sum over the numbers $h$ that are denominators of the convergents of $1/pi$ and the sum over the rest.
        $endgroup$
        – Jarek Kuben
        May 10 '17 at 21:32












      • $begingroup$
        @JarekKuben If we really can avoid using irrationality measure of $pi$, please post your entire solution separately as an answer.
        $endgroup$
        – i707107
        May 11 '17 at 0:24














      • 5




        $begingroup$
        Wow. You need a lot of sledgehammers to crack this nut.
        $endgroup$
        – marty cohen
        Mar 30 '17 at 0:39






      • 1




        $begingroup$
        @martycohen I will be interested to see if we can avoid them. But, I do not know any easier ways by now.
        $endgroup$
        – i707107
        Mar 30 '17 at 0:54






      • 1




        $begingroup$
        @i707107 Your solution is absolutely brilliant.
        $endgroup$
        – Sanjoy The Manjoy
        Mar 31 '17 at 4:36










      • $begingroup$
        Yes, we can avoid using the bound on the irrationality measure of $pi$ by splitting the sum in Erdős–Turán into the sum over the numbers $h$ that are denominators of the convergents of $1/pi$ and the sum over the rest.
        $endgroup$
        – Jarek Kuben
        May 10 '17 at 21:32












      • $begingroup$
        @JarekKuben If we really can avoid using irrationality measure of $pi$, please post your entire solution separately as an answer.
        $endgroup$
        – i707107
        May 11 '17 at 0:24








      5




      5




      $begingroup$
      Wow. You need a lot of sledgehammers to crack this nut.
      $endgroup$
      – marty cohen
      Mar 30 '17 at 0:39




      $begingroup$
      Wow. You need a lot of sledgehammers to crack this nut.
      $endgroup$
      – marty cohen
      Mar 30 '17 at 0:39




      1




      1




      $begingroup$
      @martycohen I will be interested to see if we can avoid them. But, I do not know any easier ways by now.
      $endgroup$
      – i707107
      Mar 30 '17 at 0:54




      $begingroup$
      @martycohen I will be interested to see if we can avoid them. But, I do not know any easier ways by now.
      $endgroup$
      – i707107
      Mar 30 '17 at 0:54




      1




      1




      $begingroup$
      @i707107 Your solution is absolutely brilliant.
      $endgroup$
      – Sanjoy The Manjoy
      Mar 31 '17 at 4:36




      $begingroup$
      @i707107 Your solution is absolutely brilliant.
      $endgroup$
      – Sanjoy The Manjoy
      Mar 31 '17 at 4:36












      $begingroup$
      Yes, we can avoid using the bound on the irrationality measure of $pi$ by splitting the sum in Erdős–Turán into the sum over the numbers $h$ that are denominators of the convergents of $1/pi$ and the sum over the rest.
      $endgroup$
      – Jarek Kuben
      May 10 '17 at 21:32






      $begingroup$
      Yes, we can avoid using the bound on the irrationality measure of $pi$ by splitting the sum in Erdős–Turán into the sum over the numbers $h$ that are denominators of the convergents of $1/pi$ and the sum over the rest.
      $endgroup$
      – Jarek Kuben
      May 10 '17 at 21:32














      $begingroup$
      @JarekKuben If we really can avoid using irrationality measure of $pi$, please post your entire solution separately as an answer.
      $endgroup$
      – i707107
      May 11 '17 at 0:24




      $begingroup$
      @JarekKuben If we really can avoid using irrationality measure of $pi$, please post your entire solution separately as an answer.
      $endgroup$
      – i707107
      May 11 '17 at 0:24











      2












      $begingroup$

      $color{red}{text{Not an answer, just an idea needing more work}}$ :





      I'd say $sum_n frac{(-1)^n}{n} b_n$ converges whenever $Delta^k b(n) = O(w^{k}),w< 2$ where $Delta^k b(n)$ is the $k$th forward difference, here $b(n) = |sin n|$



      If you sum by parts $k$ times, using that $sum_{n=1}^N frac{(-1)^n}{n} = frac{(-1)^N}{2 N}+ O(frac{1}{2 N^2})$ you'll get a main term $2^{-k} sum_{n=1}^{N-k} frac{(-1)^n}{n} Delta^k b(n)$





      use that $A(2M)=sum_{n=M}^infty frac{1}{(2n+1)(2n+2)} = sum_{n=m}^infty frac{1}{(2n+1)^2}-frac{1}{(2n+1)^2(2n+2)}$ and approximate with $int_{2M}^infty frac{dx}{x^2}-int_{2M}^infty frac{dx}{x^3}$ to obtain



      $$A(N) = ln 2+sum_{n=1}^N frac{(-1)^n}{n}=frac{(-1)^N}{2N}+O(frac{1}{2 N^2})$$



      Summing by parts
      $$sum_{n=1}^N frac{(-1)^n}{n} |sin n| = A(N)|sin(N)|+sum_{n=1}^{N-1} A(n) (|sin n| - |sin (n +1)|)$$



      The problematic term is $sum_{n=1}^{N-1} frac{(-1)^N}{2N} (|sin n| - |sin (n +1)|) $
      that we can sum by parts again to get a new problematic term
      $$sum_{n=1}^{N-2} frac{(-1)^n}{4n} Delta^2 b(n)$$
      where $Delta^2 b(n)=(|sin n| - |sin( n +1)|)-(|sin( n+1)| - |sin( n +2)|)$



      summing by parts $k$ times we'll have
      $$frac{1}{2^k}sum_{n=1}^{N-k} frac{(-1)^n}{n} Delta^k b(n)$$
      Where $Delta^k b(n)$ is the $k$th forward difference of $b(n) = |sin n|$






      share|cite|improve this answer











      $endgroup$


















        2












        $begingroup$

        $color{red}{text{Not an answer, just an idea needing more work}}$ :





        I'd say $sum_n frac{(-1)^n}{n} b_n$ converges whenever $Delta^k b(n) = O(w^{k}),w< 2$ where $Delta^k b(n)$ is the $k$th forward difference, here $b(n) = |sin n|$



        If you sum by parts $k$ times, using that $sum_{n=1}^N frac{(-1)^n}{n} = frac{(-1)^N}{2 N}+ O(frac{1}{2 N^2})$ you'll get a main term $2^{-k} sum_{n=1}^{N-k} frac{(-1)^n}{n} Delta^k b(n)$





        use that $A(2M)=sum_{n=M}^infty frac{1}{(2n+1)(2n+2)} = sum_{n=m}^infty frac{1}{(2n+1)^2}-frac{1}{(2n+1)^2(2n+2)}$ and approximate with $int_{2M}^infty frac{dx}{x^2}-int_{2M}^infty frac{dx}{x^3}$ to obtain



        $$A(N) = ln 2+sum_{n=1}^N frac{(-1)^n}{n}=frac{(-1)^N}{2N}+O(frac{1}{2 N^2})$$



        Summing by parts
        $$sum_{n=1}^N frac{(-1)^n}{n} |sin n| = A(N)|sin(N)|+sum_{n=1}^{N-1} A(n) (|sin n| - |sin (n +1)|)$$



        The problematic term is $sum_{n=1}^{N-1} frac{(-1)^N}{2N} (|sin n| - |sin (n +1)|) $
        that we can sum by parts again to get a new problematic term
        $$sum_{n=1}^{N-2} frac{(-1)^n}{4n} Delta^2 b(n)$$
        where $Delta^2 b(n)=(|sin n| - |sin( n +1)|)-(|sin( n+1)| - |sin( n +2)|)$



        summing by parts $k$ times we'll have
        $$frac{1}{2^k}sum_{n=1}^{N-k} frac{(-1)^n}{n} Delta^k b(n)$$
        Where $Delta^k b(n)$ is the $k$th forward difference of $b(n) = |sin n|$






        share|cite|improve this answer











        $endgroup$
















          2












          2








          2





          $begingroup$

          $color{red}{text{Not an answer, just an idea needing more work}}$ :





          I'd say $sum_n frac{(-1)^n}{n} b_n$ converges whenever $Delta^k b(n) = O(w^{k}),w< 2$ where $Delta^k b(n)$ is the $k$th forward difference, here $b(n) = |sin n|$



          If you sum by parts $k$ times, using that $sum_{n=1}^N frac{(-1)^n}{n} = frac{(-1)^N}{2 N}+ O(frac{1}{2 N^2})$ you'll get a main term $2^{-k} sum_{n=1}^{N-k} frac{(-1)^n}{n} Delta^k b(n)$





          use that $A(2M)=sum_{n=M}^infty frac{1}{(2n+1)(2n+2)} = sum_{n=m}^infty frac{1}{(2n+1)^2}-frac{1}{(2n+1)^2(2n+2)}$ and approximate with $int_{2M}^infty frac{dx}{x^2}-int_{2M}^infty frac{dx}{x^3}$ to obtain



          $$A(N) = ln 2+sum_{n=1}^N frac{(-1)^n}{n}=frac{(-1)^N}{2N}+O(frac{1}{2 N^2})$$



          Summing by parts
          $$sum_{n=1}^N frac{(-1)^n}{n} |sin n| = A(N)|sin(N)|+sum_{n=1}^{N-1} A(n) (|sin n| - |sin (n +1)|)$$



          The problematic term is $sum_{n=1}^{N-1} frac{(-1)^N}{2N} (|sin n| - |sin (n +1)|) $
          that we can sum by parts again to get a new problematic term
          $$sum_{n=1}^{N-2} frac{(-1)^n}{4n} Delta^2 b(n)$$
          where $Delta^2 b(n)=(|sin n| - |sin( n +1)|)-(|sin( n+1)| - |sin( n +2)|)$



          summing by parts $k$ times we'll have
          $$frac{1}{2^k}sum_{n=1}^{N-k} frac{(-1)^n}{n} Delta^k b(n)$$
          Where $Delta^k b(n)$ is the $k$th forward difference of $b(n) = |sin n|$






          share|cite|improve this answer











          $endgroup$



          $color{red}{text{Not an answer, just an idea needing more work}}$ :





          I'd say $sum_n frac{(-1)^n}{n} b_n$ converges whenever $Delta^k b(n) = O(w^{k}),w< 2$ where $Delta^k b(n)$ is the $k$th forward difference, here $b(n) = |sin n|$



          If you sum by parts $k$ times, using that $sum_{n=1}^N frac{(-1)^n}{n} = frac{(-1)^N}{2 N}+ O(frac{1}{2 N^2})$ you'll get a main term $2^{-k} sum_{n=1}^{N-k} frac{(-1)^n}{n} Delta^k b(n)$





          use that $A(2M)=sum_{n=M}^infty frac{1}{(2n+1)(2n+2)} = sum_{n=m}^infty frac{1}{(2n+1)^2}-frac{1}{(2n+1)^2(2n+2)}$ and approximate with $int_{2M}^infty frac{dx}{x^2}-int_{2M}^infty frac{dx}{x^3}$ to obtain



          $$A(N) = ln 2+sum_{n=1}^N frac{(-1)^n}{n}=frac{(-1)^N}{2N}+O(frac{1}{2 N^2})$$



          Summing by parts
          $$sum_{n=1}^N frac{(-1)^n}{n} |sin n| = A(N)|sin(N)|+sum_{n=1}^{N-1} A(n) (|sin n| - |sin (n +1)|)$$



          The problematic term is $sum_{n=1}^{N-1} frac{(-1)^N}{2N} (|sin n| - |sin (n +1)|) $
          that we can sum by parts again to get a new problematic term
          $$sum_{n=1}^{N-2} frac{(-1)^n}{4n} Delta^2 b(n)$$
          where $Delta^2 b(n)=(|sin n| - |sin( n +1)|)-(|sin( n+1)| - |sin( n +2)|)$



          summing by parts $k$ times we'll have
          $$frac{1}{2^k}sum_{n=1}^{N-k} frac{(-1)^n}{n} Delta^k b(n)$$
          Where $Delta^k b(n)$ is the $k$th forward difference of $b(n) = |sin n|$







          share|cite|improve this answer














          share|cite|improve this answer



          share|cite|improve this answer








          edited Mar 26 '17 at 3:59

























          answered Mar 26 '17 at 2:42









          reunsreuns

          21k21354




          21k21354























              0












              $begingroup$

              By using the Fourier series of $left|sin(n)right|$ the problem of showing that
              $$ sum_{ngeq 1}frac{(-1)^n left|sin nright|}{n} $$
              is convergent boils down to the problem of showing that
              $$ sum_{mgeq 1}frac{logleft|cos mright|}{4m^2-1}$$
              is convergent. The only issue is given by the values of $m$ such that $m$ is close to an odd multiple of $frac{pi}{2}$.

              On the other hand, since the irrationality measure of $pi$ is finite, even if $cos m$ is close to zero it cannot be closer than $frac{1}{m^{10}}$ for any $m$ large enough. Since
              $$ sum_{mgeq 1}frac{10log m}{4m^2-1} $$
              is absolutely convergent, the original series is convergent as well.






              share|cite|improve this answer









              $endgroup$


















                0












                $begingroup$

                By using the Fourier series of $left|sin(n)right|$ the problem of showing that
                $$ sum_{ngeq 1}frac{(-1)^n left|sin nright|}{n} $$
                is convergent boils down to the problem of showing that
                $$ sum_{mgeq 1}frac{logleft|cos mright|}{4m^2-1}$$
                is convergent. The only issue is given by the values of $m$ such that $m$ is close to an odd multiple of $frac{pi}{2}$.

                On the other hand, since the irrationality measure of $pi$ is finite, even if $cos m$ is close to zero it cannot be closer than $frac{1}{m^{10}}$ for any $m$ large enough. Since
                $$ sum_{mgeq 1}frac{10log m}{4m^2-1} $$
                is absolutely convergent, the original series is convergent as well.






                share|cite|improve this answer









                $endgroup$
















                  0












                  0








                  0





                  $begingroup$

                  By using the Fourier series of $left|sin(n)right|$ the problem of showing that
                  $$ sum_{ngeq 1}frac{(-1)^n left|sin nright|}{n} $$
                  is convergent boils down to the problem of showing that
                  $$ sum_{mgeq 1}frac{logleft|cos mright|}{4m^2-1}$$
                  is convergent. The only issue is given by the values of $m$ such that $m$ is close to an odd multiple of $frac{pi}{2}$.

                  On the other hand, since the irrationality measure of $pi$ is finite, even if $cos m$ is close to zero it cannot be closer than $frac{1}{m^{10}}$ for any $m$ large enough. Since
                  $$ sum_{mgeq 1}frac{10log m}{4m^2-1} $$
                  is absolutely convergent, the original series is convergent as well.






                  share|cite|improve this answer









                  $endgroup$



                  By using the Fourier series of $left|sin(n)right|$ the problem of showing that
                  $$ sum_{ngeq 1}frac{(-1)^n left|sin nright|}{n} $$
                  is convergent boils down to the problem of showing that
                  $$ sum_{mgeq 1}frac{logleft|cos mright|}{4m^2-1}$$
                  is convergent. The only issue is given by the values of $m$ such that $m$ is close to an odd multiple of $frac{pi}{2}$.

                  On the other hand, since the irrationality measure of $pi$ is finite, even if $cos m$ is close to zero it cannot be closer than $frac{1}{m^{10}}$ for any $m$ large enough. Since
                  $$ sum_{mgeq 1}frac{10log m}{4m^2-1} $$
                  is absolutely convergent, the original series is convergent as well.







                  share|cite|improve this answer












                  share|cite|improve this answer



                  share|cite|improve this answer










                  answered Jan 2 at 4:19









                  Jack D'AurizioJack D'Aurizio

                  292k33284674




                  292k33284674






























                      draft saved

                      draft discarded




















































                      Thanks for contributing an answer to Mathematics Stack Exchange!


                      • Please be sure to answer the question. Provide details and share your research!

                      But avoid



                      • Asking for help, clarification, or responding to other answers.

                      • Making statements based on opinion; back them up with references or personal experience.


                      Use MathJax to format equations. MathJax reference.


                      To learn more, see our tips on writing great answers.




                      draft saved


                      draft discarded














                      StackExchange.ready(
                      function () {
                      StackExchange.openid.initPostLogin('.new-post-login', 'https%3a%2f%2fmath.stackexchange.com%2fquestions%2f735283%2fconvergence-of-an-alternating-series-sum-n-geq-1-frac-1n-sin-nn%23new-answer', 'question_page');
                      }
                      );

                      Post as a guest















                      Required, but never shown





















































                      Required, but never shown














                      Required, but never shown












                      Required, but never shown







                      Required, but never shown

































                      Required, but never shown














                      Required, but never shown












                      Required, but never shown







                      Required, but never shown







                      Popular posts from this blog

                      Biblatex bibliography style without URLs when DOI exists (in Overleaf with Zotero bibliography)

                      ComboBox Display Member on multiple fields

                      Is it possible to collect Nectar points via Trainline?